LSAT and Law School Admissions Forum

Get expert LSAT preparation and law school admissions advice from PowerScore Test Preparation.

 erfrosen
  • Posts: 3
  • Joined: Mar 13, 2017
|
#33404
"Among the various models of Delta vacuum cleaners, one cannot accurately predict how effectively a particular model cleans...

The argument's conclusion is properly drawn if which one of the following is assumed?"

I got the correct answer of A, so I understand the point of the question, but am a little unsure of what the actual "conclusion" is as referred to in the question. Would it be:
["one cannot accurately predict how effectively a particular model cleans simply by determining how powerful its motor is"] or ["The efficiency of dust filtration systems various significantly, even between models of Delta vacuum cleaners equipped with identically powerful motors"]?

Thanks!!
 Charlie Melman
PowerScore Staff
  • PowerScore Staff
  • Posts: 85
  • Joined: Feb 10, 2017
|
#33423
Hi erfrosen,

The conclusion of this argument is that "one cannot accurately predict how effectively a particular model [of Delta Vacuum cleaners] cleans simply by determining how powerful its motor is."

We know this because the second sentence (the premise) provides support for the first (thus making it the conclusion). Since vacuums that have identically powerful motors can differ in another important respect (the efficiency of their dust filtration systems), then determining motor power is not sufficient to determine cleaning effectiveness. (Of course, there's a gap here, and you correctly identified that gap by choosing answer choice (A).)

Hope this helps!
 LSAT2018
  • Posts: 242
  • Joined: Jan 10, 2018
|
#50216
I just wanted to make sure why answers (A) and (B) are not Mistaken Negation/Mistaken Reversals of the stimulus. I was left with these two answers, and was wondering how to distinguish them.
 Rachael Wilkenfeld
PowerScore Staff
  • PowerScore Staff
  • Posts: 1358
  • Joined: Dec 15, 2011
|
#59140
Hi LSAT2018 and knowledgeconsumer,

Since both of you have similar sorts of questions, I will address them together. I would not look at this as a conditional stimulus. When we read the stimulus, we see a premise that the efficiency of dust filtration systems varies among Delta vacuum cleaners even with identically strong motors. The author draws the conclusion that you can't judge how efficiently the Delta vacuums clean based only on the strength of the motor. One way to think about Justify questions is by looking for a logical leap between the premise(s) and the conclusion. Here, there's a big jump from efficiency of dust filtration systems to efficiency of cleaning. So we will need to link those two ideas to show that the dust filtration system matters for the cleaning efficiency of a vacuum. Answer choice (A) does so. Answer choice (B) on the other hand tries to link the power of a motor to the power of the dust filtration system. But that isn't really relevant here, because the stimulus is talking about vacuums with identically strong motors. As to what models are included under Delta vacuum cleaners, I don't think it is really important. Both the stimulus and all the answer choices refer to the same items (Delta vacuum cleaners) consistently, so we don't have to look beyond the term to decide if any other sort of vacuum would fit.

Hope that helps!
Rachael
 lilRio
  • Posts: 16
  • Joined: Jul 05, 2020
|
#80064
Hello all,

Since this is a Justify the Conclusion question, I was able to answer this question in seconds using the Mechanistic Approach. I identified the conclusion, the second part of the first sentence, and honed in on the word "motor". Since "motor" appears in the conclusion and in the premise, it should NOT be in any of the answer choices. Only answer "A" conforms to this. Here is my question: how confident should one be that the Mechanistic Approach will work all the time for Justify questions? This seemed too easy...
 Jeremy Press
PowerScore Staff
  • PowerScore Staff
  • Posts: 1000
  • Joined: Jun 12, 2017
|
#80123
Hi lilRio,

Be a little careful with using the Mechanistic method of solving Justify questions that way. Notice the language we use to describe the Mechanistic method: "Elements that are common to the conclusion and at least one premise normally do not appear in the correct answer." Yes, it's true that we wouldn't normally expect to see the power of the motor mentioned in the correct answer (since it's mentioned in both premises and conclusion). But I want to be a little careful with that, since a Justify answer can potentially "do more" (i.e. include more information) than I expect it to.

Rather, what you should focus on is finding something unique to the conclusion (that the premises don't mention). Here, that would be "how effectively a particular model cleans" (i.e., cleaning effectiveness). Pick an answer that connects that information to something unique to the premises (that the conclusion doesn't mention). Here, that would be "efficiency of dust filtration systems" (i.e., dust filtration efficiency). The answer that ties those two pieces together in a way that proves the conclusion is answer choice A. That's the safer route to using the Mechanistic method.

I hope this helps!
 lilRio
  • Posts: 16
  • Joined: Jul 05, 2020
|
#80159
Dear Jeremy,

Thank you for your response. Yes, I have noticed that the Mechanistic approach "normally" follows the aforementioned pattern, but it does not always do so. I will be wary of this. Your response helped. :)
 demk26
  • Posts: 23
  • Joined: May 03, 2020
|
#80342
Hi PS,

Can you clarify the difference between Answer (A) and (D)?

From my understanding, it seems that Answer (A) is absolute ("how effective") and matches the conclusion in the stimulus, while Answer (D) is relative ("more effective"). Additionally, (D) mentions "for any two Delta vacuum cleaners with equally efficient dust filtration systems" - but the premise specifically says "The efficiency of dust filtration systems varies significantly."

Can you walk me through the reasoning behind eliminating Answer (D)?

Thank you!

(this question has been edited to correct a typo)
 Adam Tyson
PowerScore Staff
  • PowerScore Staff
  • Posts: 5153
  • Joined: Apr 14, 2011
|
#81168
The goal in this question , demk26, is to prove that the power of the motor does not, by itself, determine cleaning ability. Answer D actually weakens that argument by suggesting that the power of the motor might indeed be enough to make that determination, at least in some cases. Rather than proving that motor power is not definitive, it forces us to wonder what would happen if we compared two vacuums with different dust filtration systems. We don't want to be left wondering about anything! We want to know, with absolute certainty, that motor power is not, by itself, sufficient proof of cleaning ability. Only answer A proves that motor power is not the only thing that matters.
User avatar
 ashpine17
  • Posts: 321
  • Joined: Apr 06, 2021
|
#101949
why is the weaker language in A ok? it only says significant

Get the most out of your LSAT Prep Plus subscription.

Analyze and track your performance with our Testing and Analytics Package.